Diễn Đàn MathScopeDiễn Đàn MathScope
  Diễn Đàn MathScope
Ghi Danh Hỏi/Ðáp Community Lịch

Go Back   Diễn Đàn MathScope > Sơ Cấp > Lý Thuyết Số > Các Bài Toán Đã Được Giải

News & Announcements

Ngoài một số quy định đã được nêu trong phần Quy định của Ghi Danh , mọi người tranh thủ bỏ ra 5 phút để đọc thêm một số Quy định sau để khỏi bị treo nick ở MathScope nhé !

* Nội quy MathScope.Org

* Một số quy định chung !

* Quy định về việc viết bài trong diễn đàn MathScope

* Nếu bạn muốn gia nhập đội ngũ BQT thì vui lòng tham gia tại đây

* Những câu hỏi thường gặp

* Về việc viết bài trong Box Đại học và Sau đại học


 
 
Ðiều Chỉnh Xếp Bài
Prev Previous Post   Bài tiếp Next
Old 06-11-2010, 09:17 PM   #1
TKmathTKmath
+Thành Viên+
 
TKmathTKmath's Avatar
 
Tham gia ngày: Oct 2010
Đến từ: Cherry-blossoms
Bài gởi: 25
Thanks: 10
Thanked 7 Times in 5 Posts
[Thắc mắc]Về đồng dư

Giả sử x,y,z,n nguyên dương, p nguyên tố.
$x^p \equiv y (mod z), x^n \equiv y (mod z) $
$(p,n)=1 \Rightarrow \exists a,b \in \mathbb{Z}: ap+bn=1 $
Như vậy có thể suy ra rằng $x^1 \equiv x^{ap+bn} \equiv y (mod z) $ được hay ko?
[RIGHT][I][B]Nguồn: MathScope.ORG[/B][/I][/RIGHT]
 
__________________
Tôi cố định trong sân trường đơn điệu,
Lặng nhìn trên hình chiếu của giai nhân,
Thả hồn theo một tiếp tuyến thật gần,
Theo em mãi suốt đời về vô cực
TKmathTKmath is offline   Trả Lời Với Trích Dẫn
 

Bookmarks


Quuyền Hạn Của Bạn
You may not post new threads
You may not post replies
You may not post attachments
You may not edit your posts

BB code is Mở
Smilies đang Mở
[IMG] đang Mở
HTML đang Tắt

Chuyển đến


Múi giờ GMT. Hiện tại là 08:02 PM.


Powered by: vBulletin Copyright ©2000-2024, Jelsoft Enterprises Ltd.
Inactive Reminders By mathscope.org
[page compression: 96.15 k/99.73 k (3.59%)]